You are on page 1of 69

Theme: Proctology

A. Fissure in ano
B. Fistula in ano
C. Rectal prolapse
D. Juvenile polyps
E. Rectal adenoma
F. Intersphincteric abscess
G. Haemorroids

Please select the most likely underlying cause for the presentations described. Each
option may be used once, more than once or not at all.

1. A 21 year old female presents with a 24 hour history of increasingly severe ano-
rectal pain. On examination she is febrile and the skin surrounding the anus
looks normal. She did not tolerate an attempted digital rectal examination.

You answered Fissure in ano

The correct answer is Intersphincteric abscess

Theme from September 2012 Exam


The presence of fever and severe pain makes an abscess more likely than a
fissure. Although fissures may be painful they do not, in themselves, cause
fever. The usual management for this condition is examination of the ano-
rectum under general anaesthesia and drainage of the sepsis.

2. A 21 year old male presents with a 4 week history of frank, bright red, rectal
bleeding. This typically occurs post defecation into the toilet pan. He has a long
standing history of constipation and a previous fissure in ano. On examination
the skin surrounding the anus is normal and digital rectal examination is normal.

Haemorroids

Haemorroids are a common cause of bright red rectal bleeding. The bleeding is
typically painless. A history of constipation is usual and may have been
previously associated with a fissure (though this is less common). Haemorroids
are not always associated with external features and digital rectal examination is
usually unremarkable.

3. A 21 year old lady presents with a 6 month history of an offensive discharge


from the anus. She is otherwise well, but is increasingly annoyed at the need to
wear pads. On examination she has a small epithelial defect in the 5 o'clock
position, approximately 3cm from the anal verge.
Fistula in ano

Fistulas usually occur following previous ano-rectal sepsis. The discharge may
be foul smelling and troublesome. Patients should be listed for examination
under anaesthesia. Fistulas which are low and have little or no sphincter
involvement are usually laid open.

Ano rectal disease

Location: 3, 7, 11 o'clock position


Haemorrhoids Internal or external
Treatment: Conservative, Rubber band ligation,
Haemorrhoidectomy
Fissure in ano Location: midline 6 (posterior midline 90%) & 12 o'clock position.
Distal to the dentate line
Chronic fissure > 6/52: triad: Ulcer, sentinel pile, enlarged anal
papillae
Proctitis Causes: Crohn's, ulcerative colitis, Clostridium difficile
Ano rectal E.coli, staph aureus
abscess Positions: Perianal, Ischiorectal, Pelvirectal, Intersphincteric
Anal fistula Usually due to previous ano-rectal abscess
Intersphincteric, transsphincteric, suprasphincteric, and
extrasphincteric. Goodsalls rule determines location
Rectal prolapse Associated with childbirth and rectal intussceception. May be
internal or external
Pruritus ani Systemic and local causes
Anal neoplasm Squamous cell carcinoma commonest unlike adenocarcinoma in
rectum
Solitary rectal Associated with chronic straining and constipation. Histology shows
ulcer mucosal thickening, lamina propria replaced with collagen and
smooth muscle (fibromuscular obliteration)

Rectal prolapse

 Common especially in multiparous women.


 May be internal or external.
 Internal rectal prolapse can present insidiously.
 External prolapse can ulcerate and in long term impair continence.
 Diagnostic work up includes colonoscopy, defecating proctogram, ano rectal
manometry studies and if doubt exists and examination under anaesthesia.

Treatments for prolapse


 In the acute setting reduce it (covering it with sugar may reduce swelling.
 Delormes procedure which excises mucosa and plicates the rectum (high
recurrence rates) may be used for external prolapse.
 Altmeirs procedure which resects the colon via the perineal route has lower
recurrence rates but carries the risk of anastamotic leak.
 Rectopexy is an abdominal procedure in which the rectum is elevated and
usually supported at the level of the sacral promontory. Post operative
constipation may be reduced by limiting the dissection to the anterior plane
(laparoscopic ventral mesh rectopexy).

Pruritus ani

 Extremely common.
 Check not secondary to altered bowel habits (e.g. Diarrhoea)
 Associated with underlying diseases such as haemorrhoids.
 Examine to look for causes such as worms.
 Proctosigmoidoscopy to identify associated haemorrhoids and exclude cancer.
 Treatment is largely supportive and patients should avoid using perfumed
products around the area.

Fissure in ano

 Typically painful PR bleeding (bright red).


 Nearly always in the posterior midline.
 Usually solitary.

Treatment

 Stool softeners.
 Topical diltiazem (or GTN).
 If topical treatments fail then botulinum toxin should be injected.
 If botulinum toxin fails then males should probably undergo lateral internal
sphincterotomy and females and advancement flap.

Theme: Management of colonic polypoidal lesions

A. Reassure and discharge


B. Pan proctocolectomy
C. Hot biopsy
D. Snare polypectomy
E. Segmental colonic resection
F. Repeat endoscopy at 3 years
G. Repeat endoscopy at 1 year
H. Repeat endoscopy at 5 years
Please select the most appropriate management for the scenario given. Each option
may be used once, more than once or not at all.

1. A 43 year old man is investigated for altered bowel habit. At colonoscopy he is


found to have a 2cm polyp on a long stalk in the proximal sigmoid colon. The
rest of the colonoscopy is normal. It bears no macroscopic features of
malignancy.

Snare polypectomy

Polyps on long stalks are best managed by snare excision. It is important to


retrieve the polyp for histology.

2. A 60 year old lady is investigated for abdominal pain. A polyp is identified at


the proximal descending colon, three small polyps are also noted in the sigmoid
colon. The largest lesion is removed by snare polypectomy and the pathology
report states that this polyp is a low grade dysplastic adenoma measuring 3cm in
diameter. The remaining lesions are ablated using diathermy.

You answered Segmental colonic resection

The correct answer is Repeat endoscopy at 1 year

She is at high risk of malignancy and should be closely followed up.


Fulguration of polyps without histology is unhelpful.

3. A 73 year old lady is investigated for anaemia. At colonoscopy she is found to


have a flat broad based lesion in the caecum. This is biopsied and the histology
report states that these have diagnostic features of an adenoma with high grade
dysplasia.

You answered Pan proctocolectomy

The correct answer is Segmental colonic resection

This is most likely a malignancy and should be resected.

Polypectomy of flat broad lesions in the


right colon is difficult and where concern
arises a right hemicolectomy is probably
the safest option.
Colonic polyps

Colonic Polyps
May occur in isolation of greater numbers as part of the polyposis syndromes. In FAP
greater than 100 polyps are typically present. The risk of malignancy in association
with adenomas is related to size and is the order of 10% in a 1cm adenoma. Isolated
adenomas seldom give risk of symptoms (unless large and distal). Distally sited
villous lesions may produce mucous and if very large electrolyte disturbances may
occur.

Follow up of colonic polyps


Low risk

 1 or 2 adenomas <1cm. No follow up or re-colonoscopy at 5 years.

Moderate risk

 3 or 4 small adenomas or 1 adenoma >1cm. Re-scope at 3 years.

High risk

 >5 small adenomas or >3 with 1 of them >1cm. Re scope at 1 year.

From Atkins and Saunders Gut 2002 51 (suppl V:V6-V9). It is important to stratify
patients appropriately and ensure that a complete colonoscopy with good views was
performed.

Segmental resection or complete colectomy should be considered when:

1. Incomplete excision of malignant polyp


2. Malignant sessile polyp
3. Malignant pedunculated polyp with submucosal invasion
4. Polyps with poorly differentiated carcinoma
5. Familial polyposis coli
-Screening from teenager up to 40 years by 2 yearly sigmoidoscopy/colonoscopy
-Panproctocolectomy and Ileostomy or Restorative Panproctocolectomy.

Rectal polypoidal lesions may be amenable to trans anal endoscopic microsurger


Theme: Colonic resections

A. End ileostomy
B. Loop ileostomy
C. Ileo anal pouch
D. Loop colostomy
E. Pan proctocelectomy
F. Extended right hemicolectomy
G. Right hemicolectomy
H. Anterior resection
I. Anterior resection with covering loop ileostomy

Please select the most appropriate procedure from the list, each option may be used
once, more than once or not at all.

4. A 75 year old man requires resection of an obstructing carcinoma of the splenic


flexure.

You answered Loop ileostomy

The correct answer is Extended right hemicolectomy

Carcinoma of the splenic flexure requires extended right hemicolectomy. Or a


left hemicolectomy. The ileocolic anastomosis has a lower leak rate, particularly
when the bowel is obstructed.

5. A patient presenting with a large bowel obstruction from a low rectal cancer.

Loop colostomy

This patient should be defunctioned-definitive surgery should wait until staging


is completed. A loop ileostomy will not satisfactorily decompress an acutely
obstructed colon. Low rectal cancers that are obstructed should not usually be
primarily resected. The obstructed colon that would be used for anastomosis
would carry a high risk of anastomotic dehisence. In addition, as this is an
emergency presentation, staging may not be completed, an attempted resection
may therefore compromise the circumferential resection margin, with an
associated risk of local recurrence.

6. A 45 year old man presents with a carcinoma 10cm from the anal verge, he has
completed a long course of chemoradiotherapy and has achieved downstaging
with no evidence of threatened circumferential margin on MRI scanning.

You answered Loop colostomy

The correct answer is Anterior resection with covering loop ileostomy

Low rectal cancer is usually treated with a low anterior resection.


Contraindications to this include involvement of the sphincters (unlikely here)
and poor sphincter function that would lead to unsatisfactory function post
resection. Most colorectal surgeons defunction resections below the peritoneal
reflection as they have an intrinsically high risk of anastomotic leak. A loop
ileostomy provides a safe an satisfactory method of defunctioning these
patients. A contrast enema should be performed prior to stoma reversal.

Colorectal cancer treatment

Essentially this is surgical. Patients undergoing elective colonic resection are


increasingly being operated on laparoscopically and many centres now utilise
enhanced recovery programmes. These encourage prompt recovery by:

 Early mobilisation
 Judicious administration of fluid
 Carbohydrate loading drinks on day of surgery
 Early resumption of normal diet
 Avoidance of mechanical bowel preparation

In many elective cases mechanical bowel preparation can be avoided; this is


universally true for right sided colonic surgery. Controversy exists as to whether it is
needed for left sided surgery.

Which operation is best?


The operations for cancer are segmental resections based on blood supply and
lymphatic drainage. In the elective situation the following operations are
recommended:

Site of Type of resection Anastomosis Risk of


cancer leak
Right colon Right hemicolectomy Ileo-colic Low <5%
Transverse Extended right hemicolectomy Ileo-colic Low <5%
Splenic Extended right hemicolectomy Ileo-colic Low <5%
flexure
Splenic Left hemicolectomy Colo-colon 2-5%
flexure
Left colon Left hemicolectomy Colo-colon 2-5%
Sigmoid High anterior resection Colo-rectal 5%
colon
Upper rectum Anterior resection (TME) Colo-rectal 5%
Low rectum Anterior resection (Low TME) Colo-rectal 10%
(+/- Defunctioning
stoma)
Anal verge Abdomino-perineal excision of colon None n/a
and rectum

In the emergency setting where the bowel has perforated the risk of an anastomosis is
much greater, particularly when the anastomosis is colon-colon. In this situation an
end colostomy is often safer and can be reversed later. When resection of the sigmoid
colon is performed and an end colostomy is fashioned the operation is referred to as a
Hartmans procedure. Whilst left sided resections are more risky, ileo-colic
anastomoses are relatively safe even in the emergency setting and do not need to be
defunctioned.

References
A review of the diagnosis and management of colorectal cancer and a summary of the
UK National Institute of Clincial Excellence guidelines is provided in:
Poston G, et al . Diagnosis and management of colorectal cancer:summary of NICE
guidance. BMJ 2011: 343: d 6751
Which of the following statements in relation to fistula in ano is untrue?

A. High fistulae are safest treated with a seton insertion

B. Low fistulae may be laid open

C. Are typically probed with Lockhart Mummary probes

D. When discovered during incision and drainage of peri anal abscess


should always be probed to locate the internal opening

E. When complicating Crohns disease may respond to infliximab

Probing fistulae during acute sepsis is associated with a high complication rate and
should not be undertaken routinely.

Fistulas

 A fistula is defined as an abnormal connection between two epithelial


surfaces.
 There are many types ranging from Branchial fistulae in the neck to entero-
cutaneous fistulae abdominally.
 In general surgical practice the abdominal cavity generates the majority and
most of these arise from diverticular disease and Crohn's.
 As a general rule all fistulae will resolve spontaneously as long as there is no
distal obstruction. This is particularly true of intestinal fistulae.

The four types of fistulae are:

Enterocutaneous
These link the intestine to the skin. They may be high (>1L) or low output (<1L)
depending upon source. Duodenal /jejunal fistulae will tend to produce high volume,
electrolyte rich secretions which can lead to severe excoriation of the skin. Colo-
cutaneous fistulae will tend to leak faeculent material. Both fistulae may result from
the spontaneous rupture of an abscess cavity onto the skin (such as following perianal
abscess drainage) or may occur as a result of iatrogenic input. In some cases it may
even be surgically desirable e.g. mucous fistula following sub total colectomy for
colitis.

Suspect if there is excess fluid in the drain.

Enteroenteric or Enterocolic
This is a fistula that involves the large or small intestine. They may originate in a
similar manner to enterocutaneous fistulae. A particular problem with this fistula type
is that bacterial overgrowth may precipitate malabsorption syndromes. This may be
particularly serious in inflammatory bowel disease.

Enterovaginal
Aetiology as above.

Enterovesicular
This type of fistula goes to the bladder. These fistulas may result in frequent urinary
tract infections, or the passage of gas from the urethra during urination.

Management
Some rules relating to fistula management:

 They will heal provided there is no underlying inflammatory bowel disease


and no distal obstruction, so conservative measures may be the best option
 Where there is skin involvement, protect the overlying skin, often using a well
fitted stoma bag- skin damage is difficult to treat
 A high output fistula may be rendered more easily managed by the use of
octreotide, this will tend to reduce the volume of pancreatic secretions.
 Nutritional complications are common especially with high fistula (e.g. high
jejunal or duodenal) these may necessitate the use of TPN to provide
nutritional support together with the concomitant use of octreotide to reduce
volume and protect skin.
 When managing perianal fistulae surgeons should avoid probing the fistula
where acute inflammation is present, this almost always worsens outcomes.
 When perianal fistulae occur secondary to Crohn's disease the best
management option is often to drain acute sepsis and maintain that drainage
through the judicious use of setons whilst medical management is
implemented.
 Always attempt to delineate the fistula anatomy, for abscesses and fistulae that
have an intra abdominal source the use of barium and CT studies should show
a track. For perianal fistulae surgeons should recall Goodsall's rule in relation
to internal and external openin

You embark on a laparoscopic appendicectomy and find an appendix mass. There is


no free fluid and the patient has no evidence of peritonitis. Which is the best option?

A. Convert to a midline laparotomy and perform a


limited right hemicolectomy and end ileostomy

B. Convert to midline laparotomy and perform


and appendicectomy after taking down the
adhesions

C. Place a drain laparoscopically and administer


parenteral antibiotics

D. Send the patient for CT guided drainage

E. Wrap omentum around the area and avoid


drainage
Attempt conservative management for
appendix mass without peritonitis.

In the absence of overt peritonitis minimalist approach to these is safest, they can
always return to theatre in the event of clinical deterioration.

Appendicitis

History

 Peri umbilical abdominal pain (visceral stretching of appendix lumen and


appendix is mid gut structure) radiating to the right iliac fossa due to localised
parietal peritoneal inflammation.
 Vomit once or twice but marked and persistent vomiting is unusual.
 Diarrhoea is rare. However, pelvic appendicitis may cause localised rectal
irritation of some loose stools. A pelvic abscess may also cause diarrhoea.
 Mild pyrexia is common - temperature is usually 37.5 -38oC. Higher
temperatures are more typical of conditions like mesenteric adenitis.
 Anorexia is very common. It is very unusual for patients with appendicitis to
be hungry.

Examination

 Generalised peritonitis if perforation has occurred or localised peritonism.


 Retrocaecal appendicitis may have relatively few signs.
 Digital rectal examination may reveal boggy sensation if pelvic abscess is
present, or even tenderness with a pelvic appendix.

Diagnosis

 Typically raised inflammatory markers coupled with compatible history and


examination findings should be enough to justify appendicectomy.
 Urine analysis may show mild leucocytosis but no nitrites.
 Ultrasound is useful if females where pelvic organ pathology is suspected.
Although it is not always possible to visualise the appendix on ultrasound the
presence of free fluid (always pathological in males) should raise suspicion.
Ultrasound examination may show evidence of lumenal obstruction and thickening of
the appendiceal wall as shown below

Image sourced from Wikipedia

Treatment

 Appendicectomy which can be performed via either an open or laparoscopic


approach.
 Administration of metronidazole reduces wound infection rates.
 Patients with perforated appendicitis require copious abdominal lavage.
 Patients without peritonitis who have an appendix mass should receive broad
spectrum antibiotics and consideration given to performing an interval
appendicectomy.
 Be wary in the older patients who may have either an underlying caecal
malignancy or perforated sigmoid diverticular disease.

Laparoscopic appendicectomy is becoming increasing popular as demonstrated


below
Image sourced from Wikipedia
Theme: Colonic obstruction

A. Active observation
B. Hartmans procedure
C. Defunctioning loop colostomy
D. Colonic stent insertion
E. Sub total colectomy and ileostomy
F. Water soluble contrast enema
G. Defunctioning loop ileostomy
H. Low anterior resection

Please select the most appropriate management option from the list below. Each
option may be used once, more than once or not at all.

9. A 59 year old man presents with symptoms and signs of absolute constipation.
On investigation he is found to have large bowel obstruction and an
obstructing tumour of the distal sigmoid colon.

You answered Hartmans procedure

The correct answer is Colonic stent insertion

Colonic stents are indicated in the temporary treatment of malignant large


bowel obstruction. Lesions of the mid to low rectum are too low to stent. More
proximal lesions are more suited to this treatment modality. Complications
include perforation at the time of stent insertion.

10. A 73 year old lady is admitted with colicky abdominal pain and vomiting. On
examination she has a tense distended abdomen and some mild right iliac fossa
tenderness. Rectal examination is unremarkable. Abdominal x-ray shows
dilated large bowel with no small bowel dilatation. WCC is 15 but other blood
tests are normal. At laparotomy she has a tumour of the sigmoid colon. The
caecum appears viable, however, the bowel is dilated.

You answered Sub total colectomy and ileostomy

The correct answer is Hartmans procedure

This is safest. Some may advocate on table lavage and primary anastomosis,
this option is not on the list.

11. A 65 year old man is admitted with colicky abdominal pain and vomiting. On
examination he has a distended abdomen which is soft. Digital rectal
examination reveals a mass at the tip of the finger. Abdominal x-ray shows
dilated large bowel with no small bowel dilatation.

You answered Low anterior resection

The correct answer is Defunctioning loop colostomy

He has an obstructing rectal cancer. His obstruction should be relieved. A loop


ileostomy is less desirable here as the history suggests a competent ileocaecal
valve. The lesion would be too low for a colonic stent.

Patients with suspected large bowel


obstruction should have pseudobstruction
excluded with CT scan, gastrograffin
enema or endoscopy prior to intervention.

Colorectal cancer treatment

Essentially this is surgical. Patients undergoing elective colonic resection are


increasingly being operated on laparoscopically and many centres now utilise
enhanced recovery programmes. These encourage prompt recovery by:

 Early mobilisation
 Judicious administration of fluid
 Carbohydrate loading drinks on day of surgery
 Early resumption of normal diet
 Avoidance of mechanical bowel preparation
In many elective cases mechanical bowel preparation can be avoided; this is
universally true for right sided colonic surgery. Controversy exists as to whether it is
needed for left sided surgery.

Which operation is best?


The operations for cancer are segmental resections based on blood supply and
lymphatic drainage. In the elective situation the following operations are
recommended:

Site of Type of resection Anastomosis Risk of


cancer leak
Right colon Right hemicolectomy Ileo-colic Low <5%
Transverse Extended right hemicolectomy Ileo-colic Low <5%
Splenic Extended right hemicolectomy Ileo-colic Low <5%
flexure
Splenic Left hemicolectomy Colo-colon 2-5%
flexure
Left colon Left hemicolectomy Colo-colon 2-5%
Sigmoid High anterior resection Colo-rectal 5%
colon
Upper rectum Anterior resection (TME) Colo-rectal 5%
Low rectum Anterior resection (Low TME) Colo-rectal 10%
(+/- Defunctioning
stoma)
Anal verge Abdomino-perineal excision of colon None n/a
and rectum

In the emergency setting where the bowel has perforated the risk of an anastomosis is
much greater, particularly when the anastomosis is colon-colon. In this situation an
end colostomy is often safer and can be reversed later. When resection of the sigmoid
colon is performed and an end colostomy is fashioned the operation is referred to as a
Hartmans procedure. Whilst left sided resections are more risky, ileo-colic
anastomoses are relatively safe even in the emergency setting and do not need to be
defunctioned.

References
A review of the diagnosis and management of colorectal cancer and a summary of the
UK National Institute of Clincial Excellence guidelines is provided in:
Poston G, et al . Diagnosis and management of colorectal cancer:summary of NICE
guidance. BMJ 2011: 343: d 6751.
A 28 year old male presents with painful, bright red, rectal bleeding. On examination
he is found to have a posteriorly sited, midline, fissure in ano. What is the most
appropriate treatment?

A. Topical GTN paste

B. Sub lingual GTN paste


C. Anal stretch

D. Advancement flap

E. Tailored division of the external anal sphincter

Topical vasodilator therapy is the most commonly utilised treatment for fissure in
ano. Surgical division of the internal anal sphincter is a reasonable treatment option in
a young male. Division of the external sphincter will almost certainly result in
incontinence and is not performed. Anal stretches were associated with a high rate of
external sphincter injuries and have been discontinued for this reason.

Anal fissure

Anal fissures are a common cause of painful, bright red, rectal bleeding.
Most fissures are idiopathic and present as a painful mucocutaneous defect in the
posterior midline (90% cases). Fissures are more likely to be anteriorly located in
females, particularly if they are multiparous. Multiple fissures and those which are
located at other sites are more likely to be due to an underlying cause.
Diseases associated with fissure in ano include:

 Crohns disease
 Tuberculosis
 Internal rectal prolapse

Diagnosis
In most cases the defect can be visualised as a posterior midline epithelial defect.
Where symptoms are highly suggestive of the condition and examination findings are
unclear an examination under anaesthesia may be helpful. Atypical disease
presentation should be investigated with colonoscopy and EUA with biopsies of the
area.

Treatment
Stool softeners are important as the hard stools may tear the epithelium and result in
recurrent symptoms. The most effective first line agents are topically applied GTN
(0.2%) or Diltiazem (2%) paste. Side effects of diltiazem are better tolerated.
Resistant cases may benefit from injection of botulinum toxin or lateral internal
sphincterotomy (beware in females). Advancement flaps may be used to treat resistant
cases.
Sphincterotomy produces the best healing rates. It is associated with incontinence to
flatus in up to 10% of patients in the long term.
A 73 year old lady presents with constipation and no organic disease is identified on
investigation. Which of the following types of laxatives works by direct bowel
stimulation?

A. Magnesium sulphate
B. Lactulose

C. Potassium sodium tatrate

D. Methylcellulose

E. Sodium docusate

Laxatives

Bulk forming laxatives


Bran
Psyllium
Methylcellulose
Osmotic laxatives
Magnesium sulphate
Magnesium citrate
Sodium phosphate
Sodium sulphate
Potassium sodium tatrate
Polyethylene glycol
Stimulant laxatives
Docusates
Bisacodyl
Sodium picosulphate
Senna
Ricinoleic acid
A 53 year old man has a 2cm polyp identified and completely removed during a
colonoscopy. Histology confirms a low grade adenoma. What is the correct follow
up?

A. Discharge.

B. Repeat endoscopy in 5 years.

C. Repeat endoscopy in 3 years.

D. Segmental resection of the affected area.

E. Barium enema at 5 years.

It would be unsafe to discharge. Follow up with barium enemas for polyps is counter
intuitive.

Colonic polyps
Colonic Polyps
May occur in isolation of greater numbers as part of the polyposis syndromes. In FAP
greater than 100 polyps are typically present. The risk of malignancy in association
with adenomas is related to size and is the order of 10% in a 1cm adenoma. Isolated
adenomas seldom give risk of symptoms (unless large and distal). Distally sited
villous lesions may produce mucous and if very large electrolyte disturbances may
occur.

Follow up of colonic polyps


Low risk

 1 or 2 adenomas <1cm. No follow up or re-colonoscopy at 5 years.

Moderate risk

 3 or 4 small adenomas or 1 adenoma >1cm. Re-scope at 3 years.

High risk

 >5 small adenomas or >3 with 1 of them >1cm. Re scope at 1 year.

From Atkins and Saunders Gut 2002 51 (suppl V:V6-V9). It is important to stratify
patients appropriately and ensure that a complete colonoscopy with good views was
performed.

Segmental resection or complete colectomy should be considered when:

1. Incomplete excision of malignant polyp


2. Malignant sessile polyp
3. Malignant pedunculated polyp with submucosal invasion
4. Polyps with poorly differentiated carcinoma
5. Familial polyposis coli
-Screening from teenager up to 40 years by 2 yearly sigmoidoscopy/colonoscopy
-Panproctocolectomy and Ileostomy or Restorative Panproctocolectomy.

Rectal polypoidal lesions may be amenable to trans anal endoscopic microsurgery.


Theme: Bowel cancer management

A. Loop colostomy
B. Loop ileostomy
C. Ileo-colic bypass
D. Hartman's procedure
E. Sub total colectomy
F. Right hemicolectomy
G. Left hemicolectomy
H. Abdomino-perineal excision of the colon and rectum
I. Anterior resection

Please select the most appropriate management option for the scenario given. Each
option may be used once, more than once or not at all.

15. A 67 year old man is admitted with acute abdominal pain. He has features of
large bowel obstruction. At laparotomy he has a carcinoma of the sigmoid
colon and perforation of the caecum

You answered Loop colostomy

The correct answer is Sub total colectomy

Separate resection of right colon and sigmoid is not a good idea

16. A 89 year old lady is admitted with large bowel obstruction. She has
tenderness of the right side of her abdomen and CT scanning shows a sigmoid
lesion with liver metastasis. Her caecum measures 11cm.

You answered Ileo-colic bypass

The correct answer is Loop colostomy

A loop colostomy is the safest option. A stent would be ideal (but is not on the
list).

17. A patient has a tumour 8cm from the anal verge. Staging investigations show
localised disease only.

Anterior resection

This should be manageable with a low anterior resection

Loop colostomy remains the traditional method for


relieving inoperable large bowel obstruction.
Colonic stents are becoming increasing popular
alternatives, especially as a bridge to surgery.

Colorectal cancer treatment

Essentially this is surgical. Patients undergoing elective colonic resection are


increasingly being operated on laparoscopically and many centres now utilise
enhanced recovery programmes. These encourage prompt recovery by:

 Early mobilisation
 Judicious administration of fluid
 Carbohydrate loading drinks on day of surgery
 Early resumption of normal diet
 Avoidance of mechanical bowel preparation

In many elective cases mechanical bowel preparation can be avoided; this is


universally true for right sided colonic surgery. Controversy exists as to whether it is
needed for left sided surgery.

Which operation is best?


The operations for cancer are segmental resections based on blood supply and
lymphatic drainage. In the elective situation the following operations are
recommended:

Site of Type of resection Anastomosis Risk of


cancer leak
Right colon Right hemicolectomy Ileo-colic Low <5%
Transverse Extended right hemicolectomy Ileo-colic Low <5%
Splenic Extended right hemicolectomy Ileo-colic Low <5%
flexure
Splenic Left hemicolectomy Colo-colon 2-5%
flexure
Left colon Left hemicolectomy Colo-colon 2-5%
Sigmoid High anterior resection Colo-rectal 5%
colon
Upper rectum Anterior resection (TME) Colo-rectal 5%
Low rectum Anterior resection (Low TME) Colo-rectal 10%
(+/- Defunctioning
stoma)
Anal verge Abdomino-perineal excision of colon None n/a
and rectum

In the emergency setting where the bowel has perforated the risk of an anastomosis is
much greater, particularly when the anastomosis is colon-colon. In this situation an
end colostomy is often safer and can be reversed later. When resection of the sigmoid
colon is performed and an end colostomy is fashioned the operation is referred to as a
Hartmans procedure. Whilst left sided resections are more risky, ileo-colic
anastomoses are relatively safe even in the emergency setting and do not need to be
defunctioned.

References
A review of the diagnosis and management of colorectal cancer and a summary of the
UK National Institute of Clincial Excellence guidelines is provided in:
Poston G, et al . Diagnosis and management of colorectal cancer:summary of NICE
guidance. BMJ 2011: 343: d 6751.
Theme: Causes of rectal bleeding

A. Ulcerative colitis proctitis


B. Diversion proctitis
C. Haemorrhoidal disease
D. Fissure in ano
E. Crohns Proctitis
F. Diverticular bleed
G. Ischaemic colitis
H. Rectal intussceception

Please select the most likely cause of bleeding for the scenario given. Each option
may be used once, more than once or not at all.

18. A previously well 21 year old man is admitted with 2 week history of
diarrhoea and passage of blood and mucous rectally. He has previously
undergone an ileocaecal resection in the past for an inflammatory bowel
disorder and takes mesalazine.

Crohns Proctitis

His previous right sided resection makes crohns disease the most likely
scenario.

19. A 56 year old lady has undergone a Hartman's procedure for diverticulitis. 6
months post operatively she complains of painless passage of blood stained
mucous per rectum.

You answered Haemorrhoidal disease

The correct answer is Diversion proctitis

Rectal diversion may result in proctitis.

20. A 74 year old lady has been admitted with sudden onset profuse dark red rectal
bleeding. She was previously well. At the time of assessment her bleeding had
stopped but haemoglobin was 10.5.

You answered Ischaemic colitis

The correct answer is Diverticular bleed


This pattern of sudden onset profuse bleeding is typical of diverticular
bleeding. This often ceases spontaneously.

Rectal bleeding

Rectal bleeding is a common cause for patients to be referred to the surgical clinic. In
the clinical history it is useful to try and localise the anatomical source of the blood.
Bright red blood is usually of rectal anal canal origin, whilst dark red blood is more
suggestive of a proximally sited bleeding source. Blood which has entered the GI tract
from a gastro-duodenal source will typically resemble malaena due to the effects of
the digestive enzymes on the blood itself.

In the table below we give some typical bleeding scenarios together with physical
examination findings and causation.

Cause Type of Features in history Examination findings


bleeding
Fissure in Bright red Painful bleeding that occursMuco-epithelial defect
ano rectal post defecation in small usually in the midline
bleeding volumes. Usually antecedent posteriorly (anterior fissures
features of constipation more likely to be due to
underlying disease)
Haemorroids Bright red Post defecation bleeding Normal colon and rectum.
rectal noted both on toilet paper Proctoscopy may show
bleeding and drips into pan. May be internal haemorrhoids.
alteration of bowel habit and Internal haemorrhoids are
history of straining. No usually impalpable.
blood mixed with stool. No
local pain.
Crohns Bright red Bleeding that is Perineal inspection may show
disease or mixed accompanied by other fissures or fistulae.
blood symptoms such as altered Proctoscopy may demonstrate
bowel habit, malaise, history indurated mucosa and
of fissures (especially possibly strictures. Skip
anterior) and abscesses. lesions may be noted at
colonoscopy.
Ulcerative Bright red Diarrhoea, weight loss, Proctitis is the most marked
colitis bleeding nocturnal incontinence, finding. Peri anal disease is
often mixed passage of mucous PR. usually absent. Colonoscopy
with stool will show continuous
mucosal lesion.
Rectal cancer Bright red Alteration of bowel habit. Usually obvious mucosal
blood mixed Tenesmus may be present. abnormality. Lesion may be
volumes Symptoms of metastatic fixed or mobile depending
disease. upon disease extent.
Surrounding mucosa often
normal, although polyps may
be present.

Image showing a fissure in ano. Typically these are located posteriorly and in the
midline. Fissures at other sites may be associated with underlying disease.

Image sourced from Wikipedia

Colonoscopic image of internal haemorroids. Note these may often be impalpable.

Image sourced from Wikipedia

Investigation
 All patients presenting with rectal bleeding require digital rectal examination
and procto-sigmoidoscopy as a minimal baseline.
 Remember that haemorrhoids are typically impalpable and to attribute
bleeding to these in the absence of accurate internal inspection is
unsatisfactory.
 In young patients with no other concerning features in the history a carefully
performed sigmoidoscopy that demonstrates clear haemorrhoidal disease may
be sufficient. If clear views cannot be obtained then patients require bowel
preparation with an enema and a flexible sigmoidscopy performed.
 In those presenting with features of altered bowel habit or suspicion of
inflammatory bowel disease a colonoscopy is the best test.
 Patients with excessive pain who are suspected of having a fissure may require
an examination under general or local anaesthesia.
 In young patients with external stigmata of fissure and a compatible history it
is acceptable to treat medically and defer internal examination until the fissure
is healed. If the fissure fails to heal then internal examination becomes
necessary along the lines suggested above to exclude internal disease.

Special tests

 In patients with a malignancy of the rectum the staging investigations


comprise an MRI of the rectum to identify circumferential resection margin
compromise and to identify mesorectal nodal disease. In addition to this CT
scanning of the chest abdomen and pelvis is necessary to stage for more
distant disease. Some centres will still stage the mesorectum with endo rectal
ultrasound but this is becoming far less common.

 Patients with fissure in ano who are being considered for surgical
sphincterotomy and are females who have an obstetric history should probably
have ano rectal manometry testing performed together with endo anal
ultrasound. As this service is not universally available it is not mandatory but
in the absence of such information there are continence issues that may arise
following sphincterotomy.

Management

Disease Management
Fissure in ano GTN ointment 0.2% or diltiazem cream applied topically is the usual
first line treatment. Botulinum toxin for those who fail to respond.
Internal sphincterotomy for those who fail with botox, can be
considered at the botox stage in males.
Haemorroids Lifestyle advice, for small internal haemorrhoids can consider
injection sclerotherapy or rubber band ligation. For external
haemorrhoids consider haemorrhoidectomy. Modern options include
HALO procedure and stapled haemorrhoidectomy.
Inflammatory Medical management- although surgery may be needed for
bowel disease fistulating Crohns (setons).
Rectal cancer Anterior resection or abdomino-perineal excision of the colon and
rectum. Total mesorectal excision is now standard of care. Most
resections below the peritoneal reflection will require defunctioning
ileostomy. Most patients will require preoperative radiotherapy.

Theme: Diverticular disease management

A. Active observation
B. Colonoscopy acutely
C. Intravenous antibiotics
D. Abdominal CT Scan
E. Ultrasound scan
F. Defecating proctogram
G. Flexible sigmoidoscopy
H. Laparotomy

Please select the most appropriate immediate management for the diverticular
presentations given. Each option may be used once, more than once or not at all.

21. A 40 year old man with known diverticular disease diagnosed on colonoscopy
1 year previously is admitted with acute abdominal pain. His abdomen is
maximally tender in the left iliac fossa and he describes pneumaturia. His GP
has been giving him metronidazole for 2 days.

You answered Laparotomy

The correct answer is Abdominal CT Scan

A colovesical fistula has formed and CT will help to delineate the other
complications which may have occurred

22. An 83 year old lady with known diverticular disease is admitted with a brisk
PR bleed. On assessment the bleeding is settling and her abdomen is soft. Hb
10.2, other blood tests are normal

You answered Abdominal CT Scan

The correct answer is Active observation

Diverticular bleeds often settle spontaneously. Acute colonoscopy is rarely


helpful. She may require an elective endoscopy. Isolated diverticular bleeds
without evidence of infection do not necessarily require antibiotics.
23. A 72 year old man is admitted with large bowel obstruction and CT scan
suggests diverticular stricture in the sigmoid colon.

You answered Flexible sigmoidoscopy

The correct answer is Laparotomy

The stricture could be benign or malignant and although a lumenal study to


establish aetiology the opportunity for that intervention has passed.

70% of diverticular bleeds will settle


with conservative management.

Diverticular disease

Diverticular disease is a common surgical problem. It consists of herniation of colonic


mucosa through the muscular wall of the colon. The usual site is between the taenia
coli which vessels pierce the muscle to supply the mucosa.

Symptoms

 Altered bowel habit


 Bleeding
 Abdominal pain

Complications

 Diverticulitis
 Haemorrhage
 Development of fistula
 Perforation and faecal peritonitis
 Perforation and development of abscess
 Development of diverticular phlegmon

Diagnosis
Patients presenting in clinic will typically undergo either a colonoscopy or barium
enema as part of their diagnostic work up. Both tests will identify diverticular disease.
It can be far more difficult to confidently exclude cancer, particularly in diverticular
strictures.

Acutely unwell surgical patients should be investigated in a systematic way. Plain


abdominal films and an erect chest x-ray will identify perforation. An abdominal CT
scan with oral and intravenous contrast will help to identify whether acute
inflammation is present but also the presence of local complications such as abscess
formation.

Severity Classification- Hinchey


I Para-colonic abscess
II Pelvic abscess
III Purulent peritonitis
IV Faecal peritonitis

Treatment

 Increase dietary fibre intake.


 Mild attacks of diverticulitis may be managed conservatively with antibiotics.
 Peri colonic abscesses should be drained either surgically or radiologically.
 Recurrent episodes of acute diverticulitis requiring hospitalisation are a
relative indication for a segmental resection.
 Hinchey IV perforations (generalised faecal peritonitis) will require a
resection and usually a stoma. This group have a very high risk of post
operative complications and usually require HDU admission. Less severe
perforations may be managed by laparoscopic washout and drain insertion.

A 34 year old man presents with symptoms attributable to a fistula in ano. He is


examined in the lithotomy position and the external opening of the fistula is identified
in the 7 o'clock position. At which of the following locations is the internal opening
most likely to be identified?

A. 7 o'clock

B. 12 o'clock

C. 9 o'clock

D. 3 o'clock

E. 6 o'clock
Goodsals rule:
Anterior fistulae will tend to have an internal opening opposite the external opening.
Posterior fistulae will tend to have a curved track that passes towards the midline.

According to Goodsalls rule the track of a posteriorly sited fistula will track to the
posterior midline (i.e. 6 o'clock)

Fistulas

 A fistula is defined as an abnormal connection between two epithelial


surfaces.
 There are many types ranging from Branchial fistulae in the neck to entero-
cutaneous fistulae abdominally.
 In general surgical practice the abdominal cavity generates the majority and
most of these arise from diverticular disease and Crohn's.
 As a general rule all fistulae will resolve spontaneously as long as there is no
distal obstruction. This is particularly true of intestinal fistulae.

The four types of fistulae are:

Enterocutaneous
These link the intestine to the skin. They may be high (>1L) or low output (<1L)
depending upon source. Duodenal /jejunal fistulae will tend to produce high volume,
electrolyte rich secretions which can lead to severe excoriation of the skin. Colo-
cutaneous fistulae will tend to leak faeculent material. Both fistulae may result from
the spontaneous rupture of an abscess cavity onto the skin (such as following perianal
abscess drainage) or may occur as a result of iatrogenic input. In some cases it may
even be surgically desirable e.g. mucous fistula following sub total colectomy for
colitis.

Suspect if there is excess fluid in the drain.

Enteroenteric or Enterocolic
This is a fistula that involves the large or small intestine. They may originate in a
similar manner to enterocutaneous fistulae. A particular problem with this fistula type
is that bacterial overgrowth may precipitate malabsorption syndromes. This may be
particularly serious in inflammatory bowel disease.

Enterovaginal
Aetiology as above.

Enterovesicular
This type of fistula goes to the bladder. These fistulas may result in frequent urinary
tract infections, or the passage of gas from the urethra during urination.

Management
Some rules relating to fistula management:

 They will heal provided there is no underlying inflammatory bowel disease


and no distal obstruction, so conservative measures may be the best option
 Where there is skin involvement, protect the overlying skin, often using a well
fitted stoma bag- skin damage is difficult to treat
 A high output fistula may be rendered more easily managed by the use of
octreotide, this will tend to reduce the volume of pancreatic secretions.
 Nutritional complications are common especially with high fistula (e.g. high
jejunal or duodenal) these may necessitate the use of TPN to provide
nutritional support together with the concomitant use of octreotide to reduce
volume and protect skin.
 When managing perianal fistulae surgeons should avoid probing the fistula
where acute inflammation is present, this almost always worsens outcomes.
 When perianal fistulae occur secondary to Crohn's disease the best
management option is often to drain acute sepsis and maintain that drainage
through the judicious use of setons whilst medical management is
implemented.
 Always attempt to delineate the fistula anatomy, for abscesses and fistulae that
have an intra abdominal source the use of barium and CT studies should show
a track. For perianal fistulae surgeons should recall Goodsall's rule in relation
to internal and external openings.

Theme: Proctology

A. Haemorrhoids
B. Rectal intussceception
C. Fistula in ano
D. Fissure in ano
E. Peri-anal abscess
F. Solitary rectal ulcer
G. Marjolins ulcer

Please select the most likely disorder for the scenario given. Each option may be used
once, more than once or not at all.

25. A 38 year old lady presents with symptoms of obstructed defecation that date
back to the birth of her second child by use of ventouse. She passes mucous
and suffers from pelvic pain. Digital rectal examination and barium enema are
normal.

Rectal intussceception

Rectal intussceception (internal rectal prolapse) typically presents with


symptoms of obstructed defecation. The pathology is best demonstrated by a
defecating procotogram rather than barium enema.

26. A 23 year old male presents with bright red rectal bleeding that occurs post
defecation onto the toilet paper. He has been suffering from severe pain
associated with this. On examination there is a skin tag at 6 O'clock.

Fissure in ano

This is a typical story for fissure and should be treated with laxatives and
topical vasodilator (eg GTN) in the first instance.

27. A 19 year old male presents with bright red rectal bleeding that occurs post
defecation onto the paper and into the pan. Apart from constipation his bowel
habit is normal. Digital rectal examination is normal.

Haemorrhoids

This is likely to be haemorrhoidal disease. A sigmoidoscopy should always be


performed to exclude more sinister pathology.

Ano rectal disease

Location: 3, 7, 11 o'clock position


Haemorrhoids Internal or external
Treatment: Conservative, Rubber band ligation,
Haemorrhoidectomy
Fissure in ano Location: midline 6 (posterior midline 90%) & 12 o'clock position.
Distal to the dentate line
Chronic fissure > 6/52: triad: Ulcer, sentinel pile, enlarged anal
papillae
Proctitis Causes: Crohn's, ulcerative colitis, Clostridium difficile
Ano rectal E.coli, staph aureus
abscess Positions: Perianal, Ischiorectal, Pelvirectal, Intersphincteric
Anal fistula Usually due to previous ano-rectal abscess
Intersphincteric, transsphincteric, suprasphincteric, and
extrasphincteric. Goodsalls rule determines location
Rectal prolapse Associated with childbirth and rectal intussceception. May be
internal or external
Pruritus ani Systemic and local causes
Anal neoplasm Squamous cell carcinoma commonest unlike adenocarcinoma in
rectum
Solitary rectal Associated with chronic straining and constipation. Histology shows
ulcer mucosal thickening, lamina propria replaced with collagen and
smooth muscle (fibromuscular obliteration)

Rectal prolapse

 Common especially in multiparous women.


 May be internal or external.
 Internal rectal prolapse can present insidiously.
 External prolapse can ulcerate and in long term impair continence.
 Diagnostic work up includes colonoscopy, defecating proctogram, ano rectal
manometry studies and if doubt exists and examination under anaesthesia.

Treatments for prolapse


 In the acute setting reduce it (covering it with sugar may reduce swelling.
 Delormes procedure which excises mucosa and plicates the rectum (high
recurrence rates) may be used for external prolapse.
 Altmeirs procedure which resects the colon via the perineal route has lower
recurrence rates but carries the risk of anastamotic leak.
 Rectopexy is an abdominal procedure in which the rectum is elevated and
usually supported at the level of the sacral promontory. Post operative
constipation may be reduced by limiting the dissection to the anterior plane
(laparoscopic ventral mesh rectopexy).

Pruritus ani

 Extremely common.
 Check not secondary to altered bowel habits (e.g. Diarrhoea)
 Associated with underlying diseases such as haemorrhoids.
 Examine to look for causes such as worms.
 Proctosigmoidoscopy to identify associated haemorrhoids and exclude cancer.
 Treatment is largely supportive and patients should avoid using perfumed
products around the area.

Fissure in ano

 Typically painful PR bleeding (bright red).


 Nearly always in the posterior midline.
 Usually solitary.

Treatment

 Stool softeners.
 Topical diltiazem (or GTN).
 If topical treatments fail then botulinum toxin should be injected.
 If botulinum toxin fails then males should probably undergo lateral internal
sphincterotomy and females and advancement flap.
 Theme: Management of inflammatory bowel disease

A. Ileo-anal pouch
B. Panproctocolectomy
C. Sub total colectomy
D. Hartmans procedure
E. Right hemicolectomy
F. Intravenous steroids
G. Infliximab
H. Proctectomy

Please select the most appropriate management option from the list. Each
option may be used once, more than once or not at all.

28. A 20 year old man is admitted with bloody diarrhoea. He has been passing 10
stools per day, Hb-8.0, albumin-20. Stool culture negative. Evidence of colitis
on endoscopy. He has been on intravenous steroids for 5 days and has now
developed megacolon. His haemoglobin is falling and inflammatory markers
are static.

You answered Ileo-anal pouch

The correct answer is Sub total colectomy

This man requires a sub total colectomy. Conservative management has failed.
Patients with ulcerative colitis should undergo colectomy if there is no
significant improvement in 5-7 days after initiating medical therapy if they
have a severe attack of the disease.

29. A 19 year old lady has a long standing history of diarrhoea and weight loss.
She is investigated with an upper gastro intestinal endoscopy which is normal.
A small bowel contrast study shows a terminal ileal stricture. A colonoscopy
was performed which was normal but the endoscopist was unable to intubate
the terminal ileum. One week after the colonoscopy she is admitted with small
bowel obstruction. Steroids are administered but despite this she fails to
improve.

Right hemicolectomy

It is likely that this lady has terminal ileal disease. Although , first presentation
of Crohns disease is usually managed with IV steroids, these have been trialled
here and failed. A resection will remove the stricturing disease. If proximal
small bowel disease has not been excluded pre-operatively then this must be
evaluated "on table" during surgery to exclude other small bowel strictures.

30. A 28 year old man is reviewed in the clinic. He has suffered from Crohns
disease for many years, he has recently undergone a sub total colectomy.
However, he has residual Crohns in his rectum and this is the cause of ongoing
symptoms. Medical therapy is proving ineffective.

Proctectomy

In Crohns patients who have rectal disease and a previous sub total colectomy,
a proctectomy is the best option. An ileo-anal pouch is contra indicated in
Crohns as they may fistulate and have major post operative complications.

 IBD

Ulcerative colitis Vs Crohns

Crohn's disease Ulcerative colitis


Distribution Mouth to anus Rectum and colon
Macroscopic Cobblestone appearance, apthoid Contact bleeding
changes ulceration
Depth of Transmural inflammation Superficial inflammation
disease
Distribution Patchy Continuous
pattern
Histological Granulomas (non caseating epithelioid Crypt abscesses,
features cell aggregates with Langhans' giant Inflammatory cells in the
cells) lamina propria

Surgical treatment

Ulcerative colitis
In UC the main place for surgery is when medical treatment has failed, in the
emergency setting this will be a sub total colectomy, end ileostomy and a
mucous fistula. Electively it will be a pan proctocolectomy, an ileoanal pouch
may be a selected option for some. Remember that longstanding UC increases
colorectal cancer risk.

Image sourced from Wikipedia


Crohn's disease
Unlike UC Crohn's patients need to avoid surgeons, minimal resections are the
rule. They should not have ileoanal pouches as they will do poorly with them.
Management of Crohn's ano rectal sepsis is with a minimal approach, simply
drain sepsis and use setons to facilitate drainage. Definitive fistula surgery
should be avoided.

Image sourced from Wikipedia



A 19 year old man presents with painful rectal bleeding and is found to have an anal
fissure. Which of the following is least associated with this condition?

A. Leukaemia

B. Syphilis

C. Tuberculosis

D. Sickle cell disease

E. Crohn's disease

Anal fissures are associated with:

 Sexually transmitted diseases (syphilis, HIV)


 Inflammatory bowel disease (Crohn's up to 50%)
 Leukaemia (25% of patients)
 Tuberculosis
 Previous anal surgery

Ano rectal disease

Location: 3, 7, 11 o'clock position


Haemorrhoids Internal or external
Treatment: Conservative, Rubber band ligation,
Haemorrhoidectomy
Fissure in ano Location: midline 6 (posterior midline 90%) & 12 o'clock position.
Distal to the dentate line
Chronic fissure > 6/52: triad: Ulcer, sentinel pile, enlarged anal
papillae
Proctitis Causes: Crohn's, ulcerative colitis, Clostridium difficile
Ano rectal E.coli, staph aureus
abscess Positions: Perianal, Ischiorectal, Pelvirectal, Intersphincteric
Anal fistula Usually due to previous ano-rectal abscess
Intersphincteric, transsphincteric, suprasphincteric, and
extrasphincteric. Goodsalls rule determines location
Rectal prolapse Associated with childbirth and rectal intussceception. May be
internal or external
Pruritus ani Systemic and local causes
Anal neoplasm Squamous cell carcinoma commonest unlike adenocarcinoma in
rectum
Solitary rectal Associated with chronic straining and constipation. Histology shows
ulcer mucosal thickening, lamina propria replaced with collagen and
smooth muscle (fibromuscular obliteration)

Rectal prolapse

 Common especially in multiparous women.


 May be internal or external.
 Internal rectal prolapse can present insidiously.
 External prolapse can ulcerate and in long term impair continence.
 Diagnostic work up includes colonoscopy, defecating proctogram, ano rectal
manometry studies and if doubt exists and examination under anaesthesia.

Treatments for prolapse

 In the acute setting reduce it (covering it with sugar may reduce swelling.
 Delormes procedure which excises mucosa and plicates the rectum (high
recurrence rates) may be used for external prolapse.
 Altmeirs procedure which resects the colon via the perineal route has lower
recurrence rates but carries the risk of anastamotic leak.
 Rectopexy is an abdominal procedure in which the rectum is elevated and
usually supported at the level of the sacral promontory. Post operative
constipation may be reduced by limiting the dissection to the anterior plane
(laparoscopic ventral mesh rectopexy).

Pruritus ani

 Extremely common.
 Check not secondary to altered bowel habits (e.g. Diarrhoea)
 Associated with underlying diseases such as haemorrhoids.
 Examine to look for causes such as worms.
 Proctosigmoidoscopy to identify associated haemorrhoids and exclude cancer.
 Treatment is largely supportive and patients should avoid using perfumed
products around the area.

Fissure in ano

 Typically painful PR bleeding (bright red).


 Nearly always in the posterior midline.
 Usually solitary.

Treatment

 Stool softeners.
 Topical diltiazem (or GTN).
 If topical treatments fail then botulinum toxin should be injected.
 If botulinum toxin fails then males should probably undergo lateral internal
sphincterotomy and females and advancement flap.

A 53 year old man presents with a full thickness external rectal prolapse. Which of the
following procedures would be the most suitable surgical option?

A. Rectopexy

B. Delormes

C. Altmeirs

D. Thirsch tape

E. Abdomino-perineal excision of the rectum


Theme from 2012 Exam
As this man is young and has full thickness prolapse a rectopexy is the most
appropriate procedure. It will give the lowest recurrence rates.

Rectal prolapse

Rectal prolapse may be divided into internal and external prolapse. Patients with the
former condition may have internal intussceception of the rectum and present with
constipation, obstructed defecation and occasionally faecal incontinence. Patients with
external rectal prolapse have a full thickness external protrusion of the rectum. Risk
factors for the condition include multiparity, pelvic floor trauma and connective tissue
disorders.

Diagnosis
External prolapse is usually evident. Internal prolapse may be identified by defecating
proctography and examination under anaesthesia.
Sinister pathology should be excluded with endoscopy

Treatment

 Perineal approaches include the Delormes operation, this avoids resection and
is relatively safe but is associated with high recurrence rates. An Altmeirs
operation involves a perineal excision of the sigmoid colon and rectum, it may
be a more effective procedure than a Delormes but carries the risk of
anastomotic leak.
 Rectopexy - this is an abdominal procedure. The rectum is mobilised and fixed
onto the sacral promotary. A prosthetic mesh may be inserted. The recurrence
rates are low and the procedure is well tolerated (particularly if performed
laparoscopically).
 Thirsch tape- this is a largely historical procedure and involves encircling the
rectum with tape or wire. It may be of use in a palliative setting.

A patient has an appendicectomy and a 1.2cm carcinoid tumour is identified in the tip
of the appendix. What is the most appropriate management?

A. Watchful waiting

B. Discharge

C. Right hemicolectomy

D. Limited ileocaecal resection

E. Radioisotope scan

Individuals with small carcinoids can be discharged (<2cm and limited to the
appendix). Larger tumours should have a radioisotope scan. Where the resection
margin is positive or where the isotope scan suggests lymphatic metastasis a right
hemicolectomy should be performed.

Carcinoid syndrome

 Carcinoid tumours secrete serotonin


 Originate in neuroendocrine cells mainly in the intestine (midgut-distal
ileum/appendix)
 Can occur in the rectum, bronchi
 Hormonal symptoms mainly occur when disease spreads outside the bowel

Clinical features
- Onset: years
- Flushing face
- Palpitations
- Tricuspid stenosis causing dyspnoea
- Asthma
- Severe diarrhoea (secretory, persists despite fasting)

Investigation
- 5-HIAA in a 24-hour urine collection
- Scintigraphy
- CT scan

Treatment

 Octreotide
 Surgical removal

Theme: Management of colonic disease

A. Reassure and discharge


B. Right hemicolectomy
C. Left hemicolectomy
D. Hot biopsy
E. Snare polypectomy
F. Single colonoscopy
G. Annual colonoscopy
H. Colonoscopy every 3-5 years
I. Panproctocolectomy

For each scenario given please select the most appropriate management option. Each
option may be used once, more than once or not at all.

3. A 25 year old male presents with altered bowel habit. He is known to have
familial polyposis coli. A colonoscopy shows widespread polyps, with high
grade dysplasia in a polyp removed from the rectum.

Panproctocolectomy

There is a high risk of conversion to malignancy, therefore panproctocolectomy


is the safest option.

4. A 19 year old female presents with colicky abdominal pain, bloating and
alternating constipation/diarrhoea. Her grandmother died from colon cancer at
the age of 77 years. A digital rectal examination and general physical
examination are normal.

You answered Single colonoscopy

The correct answer is Reassure and discharge

This girl fulfills the Rome criteria for irritable bowel syndrome. Examination is
normal, therefore there is no indication for further investigation.

5. A 62 year old man is being investigated for iron deficiency anaemia. During a
colonoscopy a flat polypoidal lesion is identified in the caecum. Biopsies of this
lesion demonstrate high grade dysplasia.

You answered Colonoscopy every 3-5 years

The correct answer is Right hemicolectomy

High grade dysplasia in a flat villous lesion of the right colon is highly likely to
be associated with an invasive lesion at this site. Hot biopsy of right sided
colonic lesions is unwise an snare polypectomy would be unlikely to remove the
entire lesion.

Colonic polyps

Colonic Polyps
May occur in isolation of greater numbers as part of the polyposis syndromes. In FAP
greater than 100 polyps are typically present. The risk of malignancy in association
with adenomas is related to size and is the order of 10% in a 1cm adenoma. Isolated
adenomas seldom give risk of symptoms (unless large and distal). Distally sited
villous lesions may produce mucous and if very large electrolyte disturbances may
occur.
Follow up of colonic polyps
Low risk

 1 or 2 adenomas <1cm. No follow up or re-colonoscopy at 5 years.

Moderate risk

 3 or 4 small adenomas or 1 adenoma >1cm. Re-scope at 3 years.

High risk

 >5 small adenomas or >3 with 1 of them >1cm. Re scope at 1 year.

From Atkins and Saunders Gut 2002 51 (suppl V:V6-V9). It is important to stratify
patients appropriately and ensure that a complete colonoscopy with good views was
performed.

Segmental resection or complete colectomy should be considered when:

1. Incomplete excision of malignant polyp


2. Malignant sessile polyp
3. Malignant pedunculated polyp with submucosal invasion
4. Polyps with poorly differentiated carcinoma
5. Familial polyposis coli
-Screening from teenager up to 40 years by 2 yearly sigmoidoscopy/colonoscopy
-Panproctocolectomy and Ileostomy or Restorative Panproctocolectomy.

Rectal polypoidal lesions may be amenable to trans anal endoscopic microsurgery.


heme: Fistula management

A. No further action needed


B. Intravenous fluids
C. Intravenous fluids and nasogastric tube
D. Total parenteral nutrition and octreotide
E. Defunctioning stoma
F. Insertion of seton
G. Intravenous octreotide
H. Lay open fistula

What is the best management for the following types of fistula? Each option may be
used once, more than once or not at all.

6. A 45 year old man develops a colocutaneous fistulae following reversal of a


loop colostomy fashioned for the defunctioning of an anterior resection. Pre-
operative gastrograffin enema showed no distal obstruction or anastamotic
stricture.

You answered Lay open fistula

The correct answer is No further action needed

Colocutaneous fistulae may occur as a result of anastomotic leakage following


loop colostomy reversal. In the absence of abdominal signs a laparotomy is not
necessarily required. Signs of wound sepsis may require antibiotics. Because
there is not any distal obstruction (note normal pre-operative gastrograffin
enema) these fistulae will usually close spontaneously.

7. A 43 year old man has suffered from small bowel Crohns disease for 15 years.
Following a recent stricturoplasty he develops an enterocutaneous fistula which
is high output. Small bowel follow through shows it to be 15 cm from the DJ
flexure. His overlying skin is becoming excoriated.

You answered Lay open fistula

The correct answer is Total parenteral nutrition and octreotide

This man has a high output and anatomically high fistula. Drying up the fistula
with octreotide will not suffice, his nutrition is compromised and TPN will help.

8. A 33 year old lady presented with jaundice secondary to common bile duct
stones. A cholecystectomy and common bile duct exploration is performed and
the bile duct closed over a T tube. Six weeks post operatively a T tube
cholangiogram is performed and shows no residual stones. The T tube is
removed and five hours after removal a small amount of bile is noted to be
draining from the T tube site.

No further action needed

When the bile duct is closed over a T Tube the latex in the T tube encourages
tract fibrosis. This actually encourages a fistula to develop. The result is that
when the tube is removed any bile which leaks will usually drain through the
tract. Provided that there are no residual stones in the duct the fistula will slowly
close. Persistent high volume drainage may be managed with ERCP and
sphincterotomy.

Fistulas are more likely to heal in the


absence of distal lumenal obstruction
Fistulas

 A fistula is defined as an abnormal connection between two epithelial


surfaces.
 There are many types ranging from Branchial fistulae in the neck to entero-
cutaneous fistulae abdominally.
 In general surgical practice the abdominal cavity generates the majority and
most of these arise from diverticular disease and Crohn's.
 As a general rule all fistulae will resolve spontaneously as long as there is no
distal obstruction. This is particularly true of intestinal fistulae.

The four types of fistulae are:

Enterocutaneous
These link the intestine to the skin. They may be high (>1L) or low output (<1L)
depending upon source. Duodenal /jejunal fistulae will tend to produce high volume,
electrolyte rich secretions which can lead to severe excoriation of the skin. Colo-
cutaneous fistulae will tend to leak faeculent material. Both fistulae may result from
the spontaneous rupture of an abscess cavity onto the skin (such as following perianal
abscess drainage) or may occur as a result of iatrogenic input. In some cases it may
even be surgically desirable e.g. mucous fistula following sub total colectomy for
colitis.

Suspect if there is excess fluid in the drain.

Enteroenteric or Enterocolic
This is a fistula that involves the large or small intestine. They may originate in a
similar manner to enterocutaneous fistulae. A particular problem with this fistula type
is that bacterial overgrowth may precipitate malabsorption syndromes. This may be
particularly serious in inflammatory bowel disease.

Enterovaginal
Aetiology as above.

Enterovesicular
This type of fistula goes to the bladder. These fistulas may result in frequent urinary
tract infections, or the passage of gas from the urethra during urination.

Management
Some rules relating to fistula management:

 They will heal provided there is no underlying inflammatory bowel disease


and no distal obstruction, so conservative measures may be the best option
 Where there is skin involvement, protect the overlying skin, often using a well
fitted stoma bag- skin damage is difficult to treat
 A high output fistula may be rendered more easily managed by the use of
octreotide, this will tend to reduce the volume of pancreatic secretions.
 Nutritional complications are common especially with high fistula (e.g. high
jejunal or duodenal) these may necessitate the use of TPN to provide
nutritional support together with the concomitant use of octreotide to reduce
volume and protect skin.
 When managing perianal fistulae surgeons should avoid probing the fistula
where acute inflammation is present, this almost always worsens outcomes.
 When perianal fistulae occur secondary to Crohn's disease the best
management option is often to drain acute sepsis and maintain that drainage
through the judicious use of setons whilst medical management is
implemented.
 Always attempt to delineate the fistula anatomy, for abscesses and fistulae that
have an intra abdominal source the use of barium and CT studies should show
a track. For perianal fistulae surgeons should recall Goodsall's rule in relation
to internal and external openings.

Theme: Surgery for inflammatory bowel disease

A. Proctectomy
B. Anterior resection
C. Panproctocolectomy
D. Panproctocolectomy and ileoanal pouch
E. Sub total colectomy
F. Right hemicolectomy

Please select the most appropriate surgical modality for treating the inflammatory
bowel disease scenarios described. Each option may be used once, more than once or
not at all.

9. A 22 year old man presents with his first presentation of ulcerative colitis.
Despite aggressive medical management with steroids, azathioprine and
infliximab his symptoms remain unchanged and he has developed a
megacolon.

You answered Panproctocolectomy

The correct answer is Sub total colectomy

In patients with fulminant UC a sub total colectomy is the safest treatment


option. The rectum will be left in situ as resection of the rectum in these
acutely unwell patients carries an extremely high risk of complications.

10. A 22 year old lady has a long history of severe perianal Crohns disease with
multiple fistulae. She is keen to avoid a stoma. However, she has progressive
disease and multiple episodes of rectal bleeding. A colonoscopy shows rectal
disease only and a small bowel study shows no involvement with Crohns.
Proctectomy

Severe rectal Crohns that has developed complications such as haemorrhage


and multiple fistulae is usually best managed with proctectomy. Although a
diverting stoma may reduce the risk of local sepsis it is unlikely to reduce the
bleeding. She is keen to conserve a rectum, however, an ileoanal pouch in this
setting is unwise.

11. A 22 year old man has a long history of ulcerative colitis. His symptoms are
well controlled with steroids. However, attempts at steroid weaning and use of
steroid sparing drugs have repeatedly failed. He wishes to avoid a permanent
stoma.

Panproctocolectomy and ileoanal pouch

In patients with UC where medical management is not successful, surgical


resection may offer a chance of cure. Those patients wishing to avoid a
permanent stoma may be considered for an ileoanal pouch. However, this
procedure is only offered in the elective setting.

Surgery for inflammatory bowel disease

Patients with inflammatory bowel disease (UC and Crohns) frequently present in
surgical practice. Ulcerative colitis may be cured by surgical resection
(Proctocolectomy), this is not the case in Crohns disease which may recur and affect
other areas of the gastrointestinal tract.

Ulcerative colitis

 Elective indications for surgery include disease that is requiring maximal


therapy, or prolonged courses of steroids.
 Longstanding UC is associated with a risk of malignant transformation.
Dysplastic transformation of the colonic epithelium with associated mass
lesions is an absolute indication for a proctocolectomy.
 Emergency presentations of poorly controlled colitis that fails to respond to
medical therapy should usually be managed with a sub total colectomy.
Excision of the rectum is a procedure with a higher morbidity and is not
generally performed in the emergency setting. An end ileostomy is usually
created and the rectum either stapled off and left in situ, or, if the bowel is
very oedematous, may be brought to the surface as a mucous fistula.
 Patients with IBD have a high incidence of DVT and appropriate
thromboprophylaxis is mandatory.
 Restorative options in UC include an ileoanal pouch. This procedure can only
be performed whilst the rectum is in situ and cannot usually be undertaken as a
delayed procedure following proctectomy.
 Ileoanal pouch complications include, anastomotic dehiscence, pouchitis and
poor physiological function with seepage and soiling.

Crohns disease

 Surgical resection of Crohns disease does not equate with cure, but may
produce substantial symptomatic improvement.
 Indications for surgery include complications such as fistulae, abscess
formation and strictures.
 Extensive small bowel resections may result in short bowel syndrome and
localised stricturoplasty may allow preservation of intestinal length.
 Staging of Crohns will usually involve colonoscopy and a small bowel study
(e.g. MRI enteroclysis).
 Complex perianal fistulae are best managed with long term draining seton
sutures, complex attempts at fistula closure e.g. advancement flaps, may be
complicated by non healing and fistula recurrence.
 Severe perianal and / or rectal Crohns may require proctectomy, ileoanal
pouch reconstruction in Crohns carries a high risk of fistula formation and
pouch failure and is not recommended.
 Terminal ileal Crohns remains the commonest disease site and these patients
may be treated with limited ileocaecal resections.
 Terminal ileal Crohns may affect enterohepatic bile salt recycling and increase
the risk of gallstones.
 A 55 year old man develops an acute colonic pseudo-obstruction following a
laminectomy. Despite correction of his electrolytes and ongoing supportive
care he fails to settle. Which of the drugs listed below may improve the
situation?

A. Buscopan

B. Neostigmine

C. Metoclopramide

D. Mebevrine

E. Sodium picosulphate

Neostigmine affects the degradation of acetylcholine and will therefore
stimulate both nicotinic and muscarinic receptors. It may produce
symptomatic bradycardia and should therefore only be administered in a
monitored environment. In colonic pseudo-obstruction it produces generalised
colonic contractions and its onset is usually rapid.
 Colonic pseudo-obstruction

Colonic pseudo-obstruction is characterised by the progressive and painless
dilation of the colon. The abdomen may become grossly distended and
tympanic. Unless a complication such as impending bowel necrosis or
perforation occurs, there is usually little pain.
Diagnosis involves excluding a mechanical bowel obstruction with a plain
film and contrast enema. The underlying cause is usually electrolyte
imbalance and the condition will resolve with correction of this and supportive
care.
Patients who do not respond to supportive measures should be treated with
attempted colonoscopic decompression and/ or the drug neostigmine. In rare
cases surgery may be required.
Theme: Causes of rectal bleeding

A. Fissure in ano
B. Intersphincteric abscess
C. Haemorroidal disease
D. Proctitis
E. Solitary rectal ulcer syndrome
F. Rectal cancer
G. Anal cancer

Please select the most likely cause of rectal bleeding from the list above. Each option
may be used once, more than once or not at all.

13. An 18 year old man with a previous history of constipation presents with
bright red rectal bleeding and diarrhoea. He has suffered episodes of faecal
incontinence, which have occurred randomly throughout the day and night.

You answered Haemorroidal disease

The correct answer is Proctitis

Nocturnal diarrhoea and incontinence are typical of inflammatory bowel


disease.

14. A 56 year old man presents with episodes of pruritus ani and bright red rectal
bleeding. On examination there is a mass in the ano rectal region and biopsies
confirm squamous cell cancer.

Anal cancer

These are features of anal cancer. Anal cancers arise from the cutaneous
epithelium and are therefore typically squamous cell. They are usually
sensitive to chemoradiotherapy.
15. A 19 year old man presents with bright red rectal bleeding. He has a
longstanding history of irritable bowel syndrome. At flexible sigmoidoscopy a
lesion is biopsied and reported as showing 'fibromuscular obliteration'.

Solitary rectal ulcer syndrome

This is the typical presentation of SRUS. These patients require careful


diagnostic work up to elicit the underlying cause of their altered bowel habit.
The histological appearances of solitary rectal ulcers are characteristic and
extensive collagenous deposits are often seen. This is usually termed
fibromuscular obliteration.

Rectal bleeding

Rectal bleeding is a common cause for patients to be referred to the surgical clinic. In
the clinical history it is useful to try and localise the anatomical source of the blood.
Bright red blood is usually of rectal anal canal origin, whilst dark red blood is more
suggestive of a proximally sited bleeding source. Blood which has entered the GI tract
from a gastro-duodenal source will typically resemble malaena due to the effects of
the digestive enzymes on the blood itself.

In the table below we give some typical bleeding scenarios together with physical
examination findings and causation.

Cause Type of Features in history Examination findings


bleeding
Fissure in Bright red Painful bleeding that occurs Muco-epithelial defect
ano rectal post defecation in small usually in the midline
bleeding volumes. Usually antecedent posteriorly (anterior fissures
features of constipation more likely to be due to
underlying disease)
Haemorroids Bright red Post defecation bleeding Normal colon and rectum.
rectal noted both on toilet paper Proctoscopy may show
bleeding and drips into pan. May be internal haemorrhoids.
alteration of bowel habit and Internal haemorrhoids are
history of straining. No usually impalpable.
blood mixed with stool. No
local pain.
Crohns Bright red Bleeding that is Perineal inspection may show
disease or mixed accompanied by other fissures or fistulae.
blood symptoms such as altered Proctoscopy may demonstrate
bowel habit, malaise, history indurated mucosa and
of fissures (especially possibly strictures. Skip
anterior) and abscesses. lesions may be noted at
colonoscopy.
Ulcerative Bright red Diarrhoea, weight loss, Proctitis is the most marked
colitis bleeding nocturnal incontinence, finding. Peri anal disease is
often mixed passage of mucous PR. usually absent. Colonoscopy
with stool will show continuous
mucosal lesion.
Rectal cancer Bright red Alteration of bowel habit. Usually obvious mucosal
blood mixed Tenesmus may be present. abnormality. Lesion may be
volumes Symptoms of metastatic fixed or mobile depending
disease. upon disease extent.
Surrounding mucosa often
normal, although polyps may
be present.

Image showing a fissure in ano. Typically these are located posteriorly and in the
midline. Fissures at other sites may be associated with underlying disease.

Image sourced from Wikipedia

Colonoscopic image of internal haemorroids. Note these may often be impalpable.


Image sourced from Wikipedia

Investigation

 All patients presenting with rectal bleeding require digital rectal examination
and procto-sigmoidoscopy as a minimal baseline.
 Remember that haemorrhoids are typically impalpable and to attribute
bleeding to these in the absence of accurate internal inspection is
unsatisfactory.
 In young patients with no other concerning features in the history a carefully
performed sigmoidoscopy that demonstrates clear haemorrhoidal disease may
be sufficient. If clear views cannot be obtained then patients require bowel
preparation with an enema and a flexible sigmoidscopy performed.
 In those presenting with features of altered bowel habit or suspicion of
inflammatory bowel disease a colonoscopy is the best test.
 Patients with excessive pain who are suspected of having a fissure may require
an examination under general or local anaesthesia.
 In young patients with external stigmata of fissure and a compatible history it
is acceptable to treat medically and defer internal examination until the fissure
is healed. If the fissure fails to heal then internal examination becomes
necessary along the lines suggested above to exclude internal disease.

Special tests

 In patients with a malignancy of the rectum the staging investigations


comprise an MRI of the rectum to identify circumferential resection margin
compromise and to identify mesorectal nodal disease. In addition to this CT
scanning of the chest abdomen and pelvis is necessary to stage for more
distant disease. Some centres will still stage the mesorectum with endo rectal
ultrasound but this is becoming far less common.

 Patients with fissure in ano who are being considered for surgical
sphincterotomy and are females who have an obstetric history should probably
have ano rectal manometry testing performed together with endo anal
ultrasound. As this service is not universally available it is not mandatory but
in the absence of such information there are continence issues that may arise
following sphincterotomy.

Management

Disease Management
Fissure in ano GTN ointment 0.2% or diltiazem cream applied topically is the usual
first line treatment. Botulinum toxin for those who fail to respond.
Internal sphincterotomy for those who fail with botox, can be
considered at the botox stage in males.
Haemorroids Lifestyle advice, for small internal haemorrhoids can consider
injection sclerotherapy or rubber band ligation. For external
haemorrhoids consider haemorrhoidectomy. Modern options include
HALO procedure and stapled haemorrhoidectomy.
Inflammatory Medical management- although surgery may be needed for
bowel disease fistulating Crohns (setons).
Rectal cancer Anterior resection or abdomino-perineal excision of the colon and
rectum. Total mesorectal excision is now standard of care. Most
resections below the peritoneal reflection will require defunctioning
ileostomy. Most patients will require preoperative radiotherapy.

A 56 year old man presents with his first attack of diverticulitis. Which of the
following complications is least likely to ensue?

A. Formation of colonic strictures

B. Malignant transformation

C. Development of colovesical fistula

D. Formation of a pericolic abscess

E. Formation of a phlegmon

Diverticulitis may result in a number of complications. However, whilst malignant


disease may coincide with diverticulitis it is not, in itself, a risk factor for colonic
cancer.
Diverticular disease

Diverticular disease is a common surgical problem. It consists of herniation of colonic


mucosa through the muscular wall of the colon. The usual site is between the taenia
coli which vessels pierce the muscle to supply the mucosa.

Symptoms

 Altered bowel habit


 Bleeding
 Abdominal pain

Complications

 Diverticulitis
 Haemorrhage
 Development of fistula
 Perforation and faecal peritonitis
 Perforation and development of abscess
 Development of diverticular phlegmon

Diagnosis
Patients presenting in clinic will typically undergo either a colonoscopy or barium
enema as part of their diagnostic work up. Both tests will identify diverticular disease.
It can be far more difficult to confidently exclude cancer, particularly in diverticular
strictures.

Acutely unwell surgical patients should be investigated in a systematic way. Plain


abdominal films and an erect chest x-ray will identify perforation. An abdominal CT
scan with oral and intravenous contrast will help to identify whether acute
inflammation is present but also the presence of local complications such as abscess
formation.

Severity Classification- Hinchey


I Para-colonic abscess
II Pelvic abscess
III Purulent peritonitis
IV Faecal peritonitis

Treatment

 Increase dietary fibre intake.


 Mild attacks of diverticulitis may be managed conservatively with antibiotics.
 Peri colonic abscesses should be drained either surgically or radiologically.
 Recurrent episodes of acute diverticulitis requiring hospitalisation are a
relative indication for a segmental resection.
 Hinchey IV perforations (generalised faecal peritonitis) will require a
resection and usually a stoma. This group have a very high risk of post
operative complications and usually require HDU admission. Less severe
perforations may be managed by laparoscopic washout and drain insertion.

Theme: Cancer management

A. Right hemicolectomy and ileocolic anastomosis


B. Extended right hemicolectomy and ileocolic anastomosis
C. Left hemicolectomy and colon-colon anastomosis
D. High anterior resection and colo-rectal anastomosis
E. Anterior resection and colo-rectal anastomosis
F. Abdomino-perineal excision of colon and rectum
G. Hartman's procedure
H. Pan proctocolectomy

Please select the most appropriate operation for the scenario given. Each option may
be used once, more than once or not at all.

17. A 28 year old man with a large (>5cm) appendiceal carcinoid tumour.

Right hemicolectomy and ileocolic anastomosis

This will require formal resection.

18. A 68 year old lady has an adenocarcinoma of the rectum that invades to the
dentate line distally.

Abdomino-perineal excision of colon and rectum

Intestinal continuity cannot be restored here.

19. Emergency resection of perforated sigmoid tumour.

Hartman's procedure

Colorectal cancer treatment

Essentially this is surgical. Patients undergoing elective colonic resection are


increasingly being operated on laparoscopically and many centres now utilise
enhanced recovery programmes. These encourage prompt recovery by:
 Early mobilisation
 Judicious administration of fluid
 Carbohydrate loading drinks on day of surgery
 Early resumption of normal diet
 Avoidance of mechanical bowel preparation

In many elective cases mechanical bowel preparation can be avoided; this is


universally true for right sided colonic surgery. Controversy exists as to whether it is
needed for left sided surgery.

Which operation is best?


The operations for cancer are segmental resections based on blood supply and
lymphatic drainage. In the elective situation the following operations are
recommended:

Site of Type of resection Anastomosis Risk of


cancer leak
Right colon Right hemicolectomy Ileo-colic Low <5%
Transverse Extended right hemicolectomy Ileo-colic Low <5%
Splenic Extended right hemicolectomy Ileo-colic Low <5%
flexure
Splenic Left hemicolectomy Colo-colon 2-5%
flexure
Left colon Left hemicolectomy Colo-colon 2-5%
Sigmoid High anterior resection Colo-rectal 5%
colon
Upper rectum Anterior resection (TME) Colo-rectal 5%
Low rectum Anterior resection (Low TME) Colo-rectal 10%
(+/- Defunctioning
stoma)
Anal verge Abdomino-perineal excision of colon None n/a
and rectum

In the emergency setting where the bowel has perforated the risk of an anastomosis is
much greater, particularly when the anastomosis is colon-colon. In this situation an
end colostomy is often safer and can be reversed later. When resection of the sigmoid
colon is performed and an end colostomy is fashioned the operation is referred to as a
Hartmans procedure. Whilst left sided resections are more risky, ileo-colic
anastomoses are relatively safe even in the emergency setting and do not need to be
defunctioned.

References
A review of the diagnosis and management of colorectal cancer and a summary of the
UK National Institute of Clincial Excellence guidelines is provided in:
Poston G, et al . Diagnosis and management of colorectal cancer:summary of NICE
guidance. BMJ 2011: 343: d 6751.
A 34 year old lady presents to her general practitioner with peri anal discomfort. The
general practitioner diagnoses pruritus ani, which of the following is least associated
with the condition?

A. Hyperbilirubinaemia

B. Anal fissure

C. Leukaemia

D. Syphilis

E. Tuberculosis

Causes:

 Systemic (DM, Hyperbilirubinaemia, aplastic anaemia)


 Mechanical (diarrhoea, constipation, anal fissure)
 Infections (STDs)
 Dermatological
 Drugs (quinidine, colchicine)
 Topical agents

Ano rectal disease

Location: 3, 7, 11 o'clock position


Haemorrhoids Internal or external
Treatment: Conservative, Rubber band ligation,
Haemorrhoidectomy
Fissure in ano Location: midline 6 (posterior midline 90%) & 12 o'clock position.
Distal to the dentate line
Chronic fissure > 6/52: triad: Ulcer, sentinel pile, enlarged anal
papillae
Proctitis Causes: Crohn's, ulcerative colitis, Clostridium difficile
Ano rectal E.coli, staph aureus
abscess Positions: Perianal, Ischiorectal, Pelvirectal, Intersphincteric
Anal fistula Usually due to previous ano-rectal abscess
Intersphincteric, transsphincteric, suprasphincteric, and
extrasphincteric. Goodsalls rule determines location
Rectal prolapse Associated with childbirth and rectal intussceception. May be
internal or external
Pruritus ani Systemic and local causes
Anal neoplasm Squamous cell carcinoma commonest unlike adenocarcinoma in
rectum
Solitary rectal Associated with chronic straining and constipation. Histology shows
ulcer mucosal thickening, lamina propria replaced with collagen and
smooth muscle (fibromuscular obliteration)

Rectal prolapse

 Common especially in multiparous women.


 May be internal or external.
 Internal rectal prolapse can present insidiously.
 External prolapse can ulcerate and in long term impair continence.
 Diagnostic work up includes colonoscopy, defecating proctogram, ano rectal
manometry studies and if doubt exists and examination under anaesthesia.

Treatments for prolapse

 In the acute setting reduce it (covering it with sugar may reduce swelling.
 Delormes procedure which excises mucosa and plicates the rectum (high
recurrence rates) may be used for external prolapse.
 Altmeirs procedure which resects the colon via the perineal route has lower
recurrence rates but carries the risk of anastamotic leak.
 Rectopexy is an abdominal procedure in which the rectum is elevated and
usually supported at the level of the sacral promontory. Post operative
constipation may be reduced by limiting the dissection to the anterior plane
(laparoscopic ventral mesh rectopexy).

Pruritus ani

 Extremely common.
 Check not secondary to altered bowel habits (e.g. Diarrhoea)
 Associated with underlying diseases such as haemorrhoids.
 Examine to look for causes such as worms.
 Proctosigmoidoscopy to identify associated haemorrhoids and exclude cancer.
 Treatment is largely supportive and patients should avoid using perfumed
products around the area.

Fissure in ano

 Typically painful PR bleeding (bright red).


 Nearly always in the posterior midline.
 Usually solitary.

Treatment

 Stool softeners.
 Topical diltiazem (or GTN).
 If topical treatments fail then botulinum toxin should be injected.
 If botulinum toxin fails then males should probably undergo lateral internal
sphincterotomy and females and advancement flap.

Theme: Fistula

A. Recurrent urinary tract infection secondary to catheter


B. Nephroenteric fistula
C. Enterovesical fistula
D. High Enterocutaneous fistula
E. Low Enterocutaneous fistula
F. Enteroenteric fistula
G. Colocutaneous fistula
H. Wound infection

Please select the most likely diagnosis for the scenario given. Each option may be
used once, more than once or not at all.

21. You are the specialist trainee asked to review a 39 year old man post
gastrectomy for bleeding duodenal ulcers. He is hypotensive and tachycardic.
His drain has increased output, contains pus and has bubbles. There is
excoriated skin around the drain site.

You answered Enteroenteric fistula

The correct answer is High Enterocutaneous fistula

Suspect an enterocutaneous fistula if there is excessive drainage and bubbles.


Pus may confuse surgeons, leading them to make a diagnosis of wound
infection. If there is any uncertainty, methylene blue can be given. If
methylene blue is found in the drain, this confirms a fistula.

22. A 43 year old female presents with recurrent urinary tract infections. She
describes blood and frothy urine. She is 6 weeks post operative for a left
hemicolectomy for crohn's disease.

Enterovesical fistula

A nephroenteric fistula causes a chronic urinary tract infection, hence develops


over a longer time frame.

23. A 2 week infant has faeculent material discharging from the umbilicus.

You answered Wound infection


The correct answer is Low Enterocutaneous fistula

This baby has an enterocutaneous fistula at the umbilicus due to complete


failure of the omphalomesenteric duct to obliterate. This is treated with
resection.

Fistulas

 A fistula is defined as an abnormal connection between two epithelial


surfaces.
 There are many types ranging from Branchial fistulae in the neck to entero-
cutaneous fistulae abdominally.
 In general surgical practice the abdominal cavity generates the majority and
most of these arise from diverticular disease and Crohn's.
 As a general rule all fistulae will resolve spontaneously as long as there is no
distal obstruction. This is particularly true of intestinal fistulae.

The four types of fistulae are:

Enterocutaneous
These link the intestine to the skin. They may be high (>1L) or low output (<1L)
depending upon source. Duodenal /jejunal fistulae will tend to produce high volume,
electrolyte rich secretions which can lead to severe excoriation of the skin. Colo-
cutaneous fistulae will tend to leak faeculent material. Both fistulae may result from
the spontaneous rupture of an abscess cavity onto the skin (such as following perianal
abscess drainage) or may occur as a result of iatrogenic input. In some cases it may
even be surgically desirable e.g. mucous fistula following sub total colectomy for
colitis.

Suspect if there is excess fluid in the drain.

Enteroenteric or Enterocolic
This is a fistula that involves the large or small intestine. They may originate in a
similar manner to enterocutaneous fistulae. A particular problem with this fistula type
is that bacterial overgrowth may precipitate malabsorption syndromes. This may be
particularly serious in inflammatory bowel disease.

Enterovaginal
Aetiology as above.

Enterovesicular
This type of fistula goes to the bladder. These fistulas may result in frequent urinary
tract infections, or the passage of gas from the urethra during urination.
Management
Some rules relating to fistula management:

 They will heal provided there is no underlying inflammatory bowel disease


and no distal obstruction, so conservative measures may be the best option
 Where there is skin involvement, protect the overlying skin, often using a well
fitted stoma bag- skin damage is difficult to treat
 A high output fistula may be rendered more easily managed by the use of
octreotide, this will tend to reduce the volume of pancreatic secretions.
 Nutritional complications are common especially with high fistula (e.g. high
jejunal or duodenal) these may necessitate the use of TPN to provide
nutritional support together with the concomitant use of octreotide to reduce
volume and protect skin.
 When managing perianal fistulae surgeons should avoid probing the fistula
where acute inflammation is present, this almost always worsens outcomes.
 When perianal fistulae occur secondary to Crohn's disease the best
management option is often to drain acute sepsis and maintain that drainage
through the judicious use of setons whilst medical management is
implemented.
 Always attempt to delineate the fistula anatomy, for abscesses and fistulae that
have an intra abdominal source the use of barium and CT studies should show
a track. For perianal fistulae surgeons should recall Goodsall's rule in relation
to internal and external openings.

A 28 year old male presents with a discharging sinus in his natal cleft. He is found to
have a pilonidal sinus. Which statement is false?

A. Can occur in webs of fingers and the axilla

B. After drainage pilonidal abscesses should not be primarily closed

C. A rare complication is squamous cell carcinoma

D. In a patient with an acute abscess the Bascoms procedure is the


treatment of choice.

E. Treatment involves excising or laying open the sinus tract.


When performing incision and drainage for pilonidal abscess try to avoid making the
incision in the midline of the natal cleft.

Acute pilonidal abscesses should receive simple incision and drainage. Definitive
treatments such as a Bascoms procedure should not be undertaken when acute sepsis
is present.

Pilonidal sinus

 Occur as a result of hair debris creating sinuses in the skin (Bascom theory).
 Usually in the natal cleft of male patients after puberty.
 It is more common in Caucasians related to their hair type and growth
patterns.
 The opening of the sinus is lined by squamous epithelium, but most of its wall
consists of granulation tissue. Up to 50 cases of squamous cell carcinoma has
been described in patients with chronic pilonidal sinus disease.
 Hairs become trapped within the sinus.
 Clinically the sinus presents when acute inflammation occurs, leading to an
abscess. Patients may describe cycles of being asymptomatic and periods of
pain and discharge from the sinus.
 Treatment is difficult and opinions differ. Definitive treatment should never be
undertaken when acute infection or abscess is present as this will result in
failure.
 Definitive treatments include the Bascom procedure with excision of the pits
and obliteration of the underlying cavity. The Karydakis procedure involves
wide excision of the natal cleft such that the surface is recontoured once the
wound is closed. This avoids the shearing forces that break off the hairs and
has reasonable results.

Pilonidal sinuses are most commonly located in the midline of the natal cleft, as
illustrated below

Image sourced from Wikipedia


Theme: Colonic resections

A. Right hemicolectomy and ileocolic anastomosis


B. Extended right hemicolectomy and ileocolic anastomosis
C. Hartman's procedure
D. Defunctioning loop ileostomy only
E. Anterior resection and colo-rectal anastomosis
F. Abdomino-perineal excision of colon and rectum

Please select the most appropriate resection for the scenario given. Each option may
be used once, more than once or not at all.

25. A 58 year old man with a tumour of the splenic flexure that requires resection.

Extended right hemicolectomy and ileocolic anastomosis

Beware of the anatomy at the base of the middle colic which will require high
ligation.

26. A 63 year old man presents with a carcinoma of the upper rectum. Staging
investigations demonstrate localised disease and he is not deemed to require
and neo adjuvent therapy.

Anterior resection and colo-rectal anastomosis

Upper rectal tumours are usually amenable to restoration of intestinal


continuity and therefore an anterior resection is a reasonable treatment option.

27. A 66 year old lady presents with a tumour of the low rectum. There is a
projection inferior to within 1cm of the dentate line.

Abdomino-perineal excision of colon and rectum

Low rectal tumours such as this will require APER.

Colorectal cancer treatment

Essentially this is surgical. Patients undergoing elective colonic resection are


increasingly being operated on laparoscopically and many centres now utilise
enhanced recovery programmes. These encourage prompt recovery by:

 Early mobilisation
 Judicious administration of fluid
 Carbohydrate loading drinks on day of surgery
 Early resumption of normal diet
 Avoidance of mechanical bowel preparation
In many elective cases mechanical bowel preparation can be avoided; this is
universally true for right sided colonic surgery. Controversy exists as to whether it is
needed for left sided surgery.

Which operation is best?


The operations for cancer are segmental resections based on blood supply and
lymphatic drainage. In the elective situation the following operations are
recommended:

Site of Type of resection Anastomosis Risk of


cancer leak
Right colon Right hemicolectomy Ileo-colic Low <5%
Transverse Extended right hemicolectomy Ileo-colic Low <5%
Splenic Extended right hemicolectomy Ileo-colic Low <5%
flexure
Splenic Left hemicolectomy Colo-colon 2-5%
flexure
Left colon Left hemicolectomy Colo-colon 2-5%
Sigmoid High anterior resection Colo-rectal 5%
colon
Upper rectum Anterior resection (TME) Colo-rectal 5%
Low rectum Anterior resection (Low TME) Colo-rectal 10%
(+/- Defunctioning
stoma)
Anal verge Abdomino-perineal excision of colon None n/a
and rectum

In the emergency setting where the bowel has perforated the risk of an anastomosis is
much greater, particularly when the anastomosis is colon-colon. In this situation an
end colostomy is often safer and can be reversed later. When resection of the sigmoid
colon is performed and an end colostomy is fashioned the operation is referred to as a
Hartmans procedure. Whilst left sided resections are more risky, ileo-colic
anastomoses are relatively safe even in the emergency setting and do not need to be
defunctioned.

References
A review of the diagnosis and management of colorectal cancer and a summary of the
UK National Institute of Clincial Excellence guidelines is provided in:
Poston G, et al . Diagnosis and management of colorectal cancer:summary of NICE
guidance. BMJ 2011: 343: d 6751.
Theme: Colonic resections

A. End ileostomy
B. Loop ileostomy
C. Ileo anal pouch
D. Loop colostomy
E. Panproctocolectomy
F. Extended right hemicolectomy
G. Right hemicolectomy
H. Anterior resection
I. Anterior resection with covering loop ileostomy

Please select the most appropriate operation from the list, each option may be used
once, more than once or not at all.

28. A 75 year old man with a rectal cancer at 6 cm from the anal verge. Staging
shows localised disease.

You answered Loop colostomy

The correct answer is Anterior resection with covering loop ileostomy

Low anterior resection and covering loop ileostomy. These resections have a
5% leak rate and should be defunctioned.

29. A 65 year old man with carcinoma of the rectosigmoid junction, staging shows
localised disease.

You answered Anterior resection with covering loop ileostomy

The correct answer is Anterior resection

An anterior resection is the operation of choice. The distal resection margin


will be higher than in the first case and so will not routine require
defunctioning

30. A 29 year old man with ulcerative colitis. He has previously had a sub total
colectomy but still suffers from proctitis. He is keen to avoid a long term
stoma.

Ileo anal pouch

This man should be considered for an ileoanal pouch

Colorectal cancer treatment

Essentially this is surgical. Patients undergoing elective colonic resection are


increasingly being operated on laparoscopically and many centres now utilise
enhanced recovery programmes. These encourage prompt recovery by:
 Early mobilisation
 Judicious administration of fluid
 Carbohydrate loading drinks on day of surgery
 Early resumption of normal diet
 Avoidance of mechanical bowel preparation

In many elective cases mechanical bowel preparation can be avoided; this is


universally true for right sided colonic surgery. Controversy exists as to whether it is
needed for left sided surgery.

Which operation is best?


The operations for cancer are segmental resections based on blood supply and
lymphatic drainage. In the elective situation the following operations are
recommended:

Site of Type of resection Anastomosis Risk of


cancer leak
Right colon Right hemicolectomy Ileo-colic Low <5%
Transverse Extended right hemicolectomy Ileo-colic Low <5%
Splenic Extended right hemicolectomy Ileo-colic Low <5%
flexure
Splenic Left hemicolectomy Colo-colon 2-5%
flexure
Left colon Left hemicolectomy Colo-colon 2-5%
Sigmoid High anterior resection Colo-rectal 5%
colon
Upper rectum Anterior resection (TME) Colo-rectal 5%
Low rectum Anterior resection (Low TME) Colo-rectal 10%
(+/- Defunctioning
stoma)
Anal verge Abdomino-perineal excision of colon None n/a
and rectum

In the emergency setting where the bowel has perforated the risk of an anastomosis is
much greater, particularly when the anastomosis is colon-colon. In this situation an
end colostomy is often safer and can be reversed later. When resection of the sigmoid
colon is performed and an end colostomy is fashioned the operation is referred to as a
Hartmans procedure. Whilst left sided resections are more risky, ileo-colic
anastomoses are relatively safe even in the emergency setting and do not need to be
defunctioned.

References
A review of the diagnosis and management of colorectal cancer and a summary of the
UK National Institute of Clincial Excellence guidelines is provided in:
Poston G, et al . Diagnosis and management of colorectal cancer:summary of NICE
guidance. BMJ 2011: 343: d 6751.
Theme: Colonic resections
A. Loop colostomy
B. Loop ileostomy
C. Colonic stent
D. Hartmann's procedure
E. Sub total colectomy
F. Right hemicolectomy
G. Left hemicolectomy
H. Abdomino-perineal excision of the colon and rectum
I. Anterior resection

Please select the most appropriate management for the colonic case indicated. Each
option may be used once, more than once or not at all.

31. A 68 year old man is admitted with large bowel obstruction. On examination
he has a large tumour palpable 4cm from the anal verge. Imaging shows a
caecal diameter of 10cm and no evidence of perforation.

You answered Loop ileostomy

The correct answer is Loop colostomy

This patient needs to be defunctioned. The tumour is too low for a colonic
stent. Primary resection in this group is unwise as CRM may be involved.

32. A 72 year old lady is admitted with large bowel obstruction. On examination
she has an empty rectum. CT scan shows a 10cm caecum and a tumour present
at the rectosigmoid junction.

You answered Anterior resection

The correct answer is Colonic stent

Stenting will avoid the need for emergency surgery and once she has been
stabilised a primary resection and anastomosis performed

33. A 65 year old patient is admitted with acute abdominal pain. An erect CXR
shows free air. At laparotomy a perforated sigmoid cancer is found. There is
no evidence of metastatic disease.

Hartmann's procedure

A Hartman's procedure is standard. A primary anastomosis in this setting


would be unsafe (high risk of leak)
Rectal cancer with a threatened CRM
should be downstaged with
chemoradiotherapy prior to resection

Colorectal cancer treatment

Essentially this is surgical. Patients undergoing elective colonic resection are


increasingly being operated on laparoscopically and many centres now utilise
enhanced recovery programmes. These encourage prompt recovery by:

 Early mobilisation
 Judicious administration of fluid
 Carbohydrate loading drinks on day of surgery
 Early resumption of normal diet
 Avoidance of mechanical bowel preparation

In many elective cases mechanical bowel preparation can be avoided; this is


universally true for right sided colonic surgery. Controversy exists as to whether it is
needed for left sided surgery.

Which operation is best?


The operations for cancer are segmental resections based on blood supply and
lymphatic drainage. In the elective situation the following operations are
recommended:

Site of Type of resection Anastomosis Risk of


cancer leak
Right colon Right hemicolectomy Ileo-colic Low <5%
Transverse Extended right hemicolectomy Ileo-colic Low <5%
Splenic Extended right hemicolectomy Ileo-colic Low <5%
flexure
Splenic Left hemicolectomy Colo-colon 2-5%
flexure
Left colon Left hemicolectomy Colo-colon 2-5%
Sigmoid High anterior resection Colo-rectal 5%
colon
Upper rectum Anterior resection (TME) Colo-rectal 5%
Low rectum Anterior resection (Low TME) Colo-rectal 10%
(+/- Defunctioning
stoma)
Anal verge Abdomino-perineal excision of colon None n/a
and rectum

In the emergency setting where the bowel has perforated the risk of an anastomosis is
much greater, particularly when the anastomosis is colon-colon. In this situation an
end colostomy is often safer and can be reversed later. When resection of the sigmoid
colon is performed and an end colostomy is fashioned the operation is referred to as a
Hartmans procedure. Whilst left sided resections are more risky, ileo-colic
anastomoses are relatively safe even in the emergency setting and do not need to be
defunctioned.

References
A review of the diagnosis and management of colorectal cancer and a summary of the
UK National Institute of Clincial Excellence guidelines is provided in:
Poston G, et al . Diagnosis and management of colorectal cancer:summary of NICE
guidance. BMJ 2011: 343: d 6751.

A 23 year old lady presents with a posteriorly sited fissure in ano. Treatment with
stool softeners and topical GTN has failed to improve matters. Which of the following
would be the most appropriate next management step?

A. Lords procedure

B. Fissurectomy and injection of botulinum toxin

C. Lateral internal sphincterotomy

D. Endoanal advancement flap

E. Surgical division of the external anal sphincter

The next most appropriate management option when GTN or other topical nitrates has
failed is to consider botulinum toxin injection. In males a lateral internal
sphincterotomy would be an acceptable alternative. In a female who has yet to
conceive this may predispose to delayed increased risk of sphincter dysfunction.
Division of the external sphincter will result in faecal incontinence and is not a
justified treatment for fissure.

Rectal bleeding

Rectal bleeding is a common cause for patients to be referred to the surgical clinic. In
the clinical history it is useful to try and localise the anatomical source of the blood.
Bright red blood is usually of rectal anal canal origin, whilst dark red blood is more
suggestive of a proximally sited bleeding source. Blood which has entered the GI tract
from a gastro-duodenal source will typically resemble malaena due to the effects of
the digestive enzymes on the blood itself.

In the table below we give some typical bleeding scenarios together with physical
examination findings and causation.

Cause Type of Features in history Examination findings


bleeding
Fissure in Bright red Painful bleeding that occurs Muco-epithelial defect
ano rectal post defecation in small usually in the midline
bleeding volumes. Usually antecedent posteriorly (anterior fissures
features of constipation more likely to be due to
underlying disease)
Haemorroids Bright red Post defecation bleeding Normal colon and rectum.
rectal noted both on toilet paper Proctoscopy may show
bleeding and drips into pan. May be internal haemorrhoids.
alteration of bowel habit and Internal haemorrhoids are
history of straining. No usually impalpable.
blood mixed with stool. No
local pain.
Crohns Bright red Bleeding that is Perineal inspection may show
disease or mixed accompanied by other fissures or fistulae.
blood symptoms such as altered Proctoscopy may demonstrate
bowel habit, malaise, history indurated mucosa and
of fissures (especially possibly strictures. Skip
anterior) and abscesses. lesions may be noted at
colonoscopy.
Ulcerative Bright red Diarrhoea, weight loss, Proctitis is the most marked
colitis bleeding nocturnal incontinence, finding. Peri anal disease is
often mixed passage of mucous PR. usually absent. Colonoscopy
with stool will show continuous
mucosal lesion.
Rectal cancer Bright red Alteration of bowel habit. Usually obvious mucosal
blood mixed Tenesmus may be present. abnormality. Lesion may be
volumes Symptoms of metastatic fixed or mobile depending
disease. upon disease extent.
Surrounding mucosa often
normal, although polyps may
be present.

Image showing a fissure in ano. Typically these are located posteriorly and in the
midline. Fissures at other sites may be associated with underlying disease.
Image sourced from Wikipedia

Colonoscopic image of internal haemorroids. Note these may often be impalpable.

Image sourced from Wikipedia

Investigation

 All patients presenting with rectal bleeding require digital rectal examination
and procto-sigmoidoscopy as a minimal baseline.
 Remember that haemorrhoids are typically impalpable and to attribute
bleeding to these in the absence of accurate internal inspection is
unsatisfactory.
 In young patients with no other concerning features in the history a carefully
performed sigmoidoscopy that demonstrates clear haemorrhoidal disease may
be sufficient. If clear views cannot be obtained then patients require bowel
preparation with an enema and a flexible sigmoidscopy performed.
 In those presenting with features of altered bowel habit or suspicion of
inflammatory bowel disease a colonoscopy is the best test.
 Patients with excessive pain who are suspected of having a fissure may require
an examination under general or local anaesthesia.
 In young patients with external stigmata of fissure and a compatible history it
is acceptable to treat medically and defer internal examination until the fissure
is healed. If the fissure fails to heal then internal examination becomes
necessary along the lines suggested above to exclude internal disease.

Special tests

 In patients with a malignancy of the rectum the staging investigations


comprise an MRI of the rectum to identify circumferential resection margin
compromise and to identify mesorectal nodal disease. In addition to this CT
scanning of the chest abdomen and pelvis is necessary to stage for more
distant disease. Some centres will still stage the mesorectum with endo rectal
ultrasound but this is becoming far less common.

 Patients with fissure in ano who are being considered for surgical
sphincterotomy and are females who have an obstetric history should probably
have ano rectal manometry testing performed together with endo anal
ultrasound. As this service is not universally available it is not mandatory but
in the absence of such information there are continence issues that may arise
following sphincterotomy.

Management

Disease Management
Fissure in ano GTN ointment 0.2% or diltiazem cream applied topically is the usual
first line treatment. Botulinum toxin for those who fail to respond.
Internal sphincterotomy for those who fail with botox, can be
considered at the botox stage in males.
Haemorroids Lifestyle advice, for small internal haemorrhoids can consider
injection sclerotherapy or rubber band ligation. For external
haemorrhoids consider haemorrhoidectomy. Modern options include
HALO procedure and stapled haemorrhoidectomy.
Inflammatory Medical management- although surgery may be needed for
bowel disease fistulating Crohns (setons).
Rectal cancer Anterior resection or abdomino-perineal excision of the colon and
rectum. Total mesorectal excision is now standard of care. Most
resections below the peritoneal reflection will require defunctioning
ileostomy. Most patients will require preoperative radiotherapy.

You might also like